LSAT and Law School Admissions Forum

Get expert LSAT preparation and law school admissions advice from PowerScore Test Preparation.

 rameday
  • Posts: 94
  • Joined: May 07, 2014
|
#14659
Hello,

I am having trouble with the following games in the lesson 3 homework.

Game 8 October 1991 question 12
Game 9 june 1992 questions 2-6 - i am having difficulty with the set up
Game 10 June 1993 questions 1-5 i am having difficulty with the set up
game 11 september 2006 questions 1-5 i am having difficulty with the set up
game 12 june 1994 questions 7-11 i am having difficulty with the set up
User avatar
 KelseyWoods
PowerScore Staff
  • PowerScore Staff
  • Posts: 1079
  • Joined: Jun 26, 2013
|
#14660
Hi rameday!

It would help if you could post about each of these games separately and tell us a little more about your difficulty with the setups. Have you read through the explanations in the back of the book? If you let us know which parts of the setups you are struggling with, we can help you more specifically, rather than just regurgitating the explanations you already have in your book :)

Looking forward to helping you with these!

Best,
Kelsey
 rameday
  • Posts: 94
  • Joined: May 07, 2014
|
#14665
Hello,

So for game 10 I am having a bit of trouble understanding the inferences made with respect to the third rule with respect to K and O (3-150).

For game 11. I also don't understand why the answer key said that from the fourth rule if O cannot be chosen for stop 1 or 2, then N cannot be chosen for stops 2 or 3. (3-154)

For game 12 i am a bit confused as to why the fourth rule is conditional and why a G not law does not appear under monday

For game 5 on 3-22 question 18, I didn't quite understand the explanation on the student center. If R is performed 7th the website says then we can conclude that T is performed first. But couldn't T be performed 8th after R (if performed 7th) in accordance with rule 1?

A
 Ron Gore
PowerScore Staff
  • PowerScore Staff
  • Posts: 220
  • Joined: May 15, 2013
|
#14671
Hi A,
rameday wrote:So for game 10 I am having a bit of trouble understanding the inferences made with respect to the third rule with respect to K and O (3-150).
The key to understanding this rule is that K and O must always be on adjacent days. To get a feel for how this rule can lead to difficulty, jot down a few quick templates where you shift which day has two students, then move the K/O block in various positions on those days. I am not saying this activity would be a good idea on test day, but seeing the limitations of the K/O block in this setup with your own eyes through experimentation will make a whole lot more sense to you than any answer I would provide. This type of experimentation in practice is a great way to get a better feel for the games and how they work.
rameday wrote:For game 11. I also don't understand why the answer key said that from the fourth rule if O cannot be chosen for stop 1 or 2, then N cannot be chosen for stops 2 or 3. (3-154)


The fourth rule says that L cannot immediately precede N, and that M cannot immediately precede N. The only other airline left is O. As a result, if an airline immediately precedes N, it must be O. So, if O cannot be stop 1, then N cannot be stop 2, because no airline other than O can immediately precede N. And the same holds true for stops 2 and 3. If O cannot be in stop 2, then N cannot be in stop 3.
rameday wrote: For game 12 i am a bit confused as to why the fourth rule is conditional and why a G not law does not appear under monday
The word "never" is what makes Rule 4 conditional. A conditional relationship is absolute. Here, if both Fuentes and Jackson work, then Jackson must work earlier in the week than Fuentes, because they cannot work on the same day and Fuentes never works later in the week than Jackson. What may have confused you is the fact that it is not necessarily the case that all of the firefighters work. So, we cannot assume that both Jackson and Fuentes will work. But, in the case that both do work, F > J.

The same consideration explains why there is not a Not Law for G under Monday. We cannot be certain that Howell works, and the limitation on G not working on Monday is required only if Howell works.
rameday wrote:For game 5 on 3-22 question 18, I didn't quite understand the explanation on the student center. If R is performed 7th the website says then we can conclude that T is performed first. But couldn't T be performed 8th after R (if performed 7th) in accordance with rule 1?
Rule 4 says that the 8th performance is either L or H. So, it cannot be T.

Hope those answers help.

Ron

Get the most out of your LSAT Prep Plus subscription.

Analyze and track your performance with our Testing and Analytics Package.